Categories
Courses Curator's Favorites Exam Questions Lecture Notes M.I.T. Problem Sets Suggested Reading Syllabus Uncategorized Undergraduate

M.I.T. Principles of Macroeconomics. Slides, problems sets, exams. Krugman, 1998

 

One might think that putting together robust links to economics course materials found in the internet archive, Wayback Machine, would be relatively straightforward, and sometimes it is. But most of us are inconsistent with the use of folders and sometimes pages get updated by other people so that traditional archival persistence is generally required to find missing pieces to the historical puzzle. In any event, today’s post manages to pack links to course content for a principles of macroeconomics course taught at M.I.T. exactly two decades ago by Paul Krugman.

I remember that semester well, because immediately after Paul Krugman finished his teaching obligations at M.I.T. for that fall term, he came to Berlin to receive an honorary doctorate from Freie Universität Berlin. The audio recording to his lecture “The return of demand-side economics” can still be heard (beginning around minute 2:00) at a webpage maintained by the John-F.-Kennedy Institute for North American Studies of Freie Universität.

_________________

14.02 Principles of Macroeconomics
Fall 1998
Professor Paul Krugman

Course Syllabus

Text: Olivier Blanchard, Macroeconomics.

Schedule (with links to lecture slides and exams)

Note: the lecture slides may differ slightly from those presented in class.

September 14 — Chapter 2: Preliminary Concepts

Slides: Tracking the Macroeconomy: Five Key Aggregates

September 16 — Chapter 3 & 4: The Goods Market (lecture by Roberto Rigobón)

September 21 — Chapter 5: Financial Markets

Slides: Review. Multiplier Analysis

Handout by Adam B. Ashcroft on bond yields about here

September 23 — Chapter 5: More on Financial Markets

Slides: The Federal Reserve and the Money Supply

September 28 — Chapter 6: IS-LM

Slides: The IS-LM Model

September 30 — Chapter 7: Expectations

Slides: Expectations and Macroeconomics

October 5 — Chapter 8: Expectations, Consumption, and Investment

Slides: Consumer Behavior–Not that simple

October 7 — Banks and the Banking System

Slides: Banking and the Financial System

October 8
Exam 1

Exam #1 Questions
Solutions

October 13 — Chapter 9: Expectations and Financial Markets

Slides: (missing)

October 14 — Chapter 10: Expectations and Policy

Slides: Expectations and Macroeconomic Policy 

October 19 — Chapter 11: Introduction to the Open Economy

Slides: The Open Economy

October 21 — Chapter 12: The Open Economy Goods Market

Slides: Macroeconomics in the Open Economy

October 26 — Chapter 13: Interest Rates and Exchange Rates

Slides: What Determines Exchange Rates

Handout on exchange rates about here.

October 28 — Chapter 13: Exchange Rate Regimes

Slides: Fixed Exchange Rates

November 2 — Chapter 14: Expectations, Crises, and General Mayheim

Slides: (missing)

November 4 — Chapter 15: The Labor Market

Slides: Why Study the Labor Market?

November 5
Exam 2

Exam #2 review
Exam #2 questions
Solutions

November 9 — Chapter 16: General Equilibrium

Slides: Putting It All Together–AS-AD

November 16 — Chapter 17: The Phillips Curve

Slides: From Aggregate Supply to the Phillips Curve

November 18 — Chapter 18: Disinflation

Slides: Long-run Unemployment-Inflation Dynamics [note: “?” for the greek letter pi, i.e. rate of inflation]

November 23 — Chapter 19 & 21: Seigniorage and Devaluation

Slides: Inflation, Interest Rates, and Hyperinflation

November 25 — Chapter 22 & 23: Long-run Growth

Slides: Economic Growth

November 30 — Chapter 24: Technical Progress

Slides: Savings, Investment, and Growth

Handout on growth about here.

December 2 — Chapter 20: Great Depression and European Unemployment

Slides: High Unemployment and Growth Slowdowns 

December 7 — Zuckerman & Krugman Foreign Affairs articles (lecture by Roberto Rigobón)

[Paul Krugman, Debate: America the Boastful, and Mortimer B. Zuckerman, Debate: A Second American Century,  Foreign Affairs (May/June 1998)]

December 9 — Course Review

Slide: Overview Graphic [Note: graphic cut-off on right hand side]

Final Examination (December, 2018)

Final Exam Review
Pablo Garcia’s Review
Final Exam Questions 

 

Problem Sets

Set Number Assigned Due Returned
1 9-11 9-18 9-21
2 9-18 9-25 9-28
3 9-25 10-2 10-5
4 10-9 10-16 10-19
5 10-16 10-23 10-26
6 10-23 10-30 11-2
7 11-6 11-13 11-16
8 11-13 11-20 11-23
9 11-20 12-4 12-7

 

Problem Set 1
Solutions

Problem Set 2
Solutions

Problem Set 3
Solutions (missing)

Optional Problem Set 1
Solutions

Problem Set 4
Solutions

Problem Set 5
Solutions

Problem Set 6
Solutions

Problem Set 7
Solutions

Optional Set 2
Solutions

Problem Set 8
Solutions

Problem Set 9
Solutions

Optional Set 3
Solutions

 

Image: Photograph taken in December 1998 at Cecilienhof, Potsdam (Germany). Irwin Collier and Paul Krugman.

Categories
Cambridge Exam Questions

Cambridge. Economics Tripos Examinations, 1921

 

The following eighteen examinations constituted the Economics Tripos at Cambridge University in 1921. Fifteen exams covered economic theory, policy, history and statistics with three exams covering politics and international law.

Previously posted Cambridge economics exams have been posted at the following links:

Guide to Moral Sciences Tripos 1891
Economics Tripos 1931
Economics Tripos 1932

___________________

ECONOMICS TRIPOS
PART I

Monday, May 30, 1921. 9–12.
GENERAL ECONOMICS. I.

  1. Comment on the following passage: “Les vérités d’économie politique pure fouruiront la solution des problèmes les plus importants, les plus débattus et les moins éclairés d’économie politique appliquée et d’économie sociale.”
  2. “Prices may be determined over a short period by demand and supply; but in the long run they are governed by the superior force, Cost of Production.” Comment on this statement.
  3. What do you understand by the term, Joint Products? Would you apply the term to the production of chocolate and cocoa by the same factory? What principles govern the prices of Joint Products?
  4. “The ordinary progress of a society, which increases in wealth, is at all times tending to augment the incomes of landlords.” Discuss this statement. What effect would you expect a great development of internal transport facilities to exert upon urban rents?
  5. Consider the relative advantages of wages settlements for any industry (a) upon a uniform national basis, (b) upon varying district bases.
  6. Compare the attempts of trade unions to raise wages with those of combinations to raise prices as regards (a) the methods which they employ, (b) the economic conditions of their success.
  7. Distinguish the various types of business which it is customary to describe as Cooperative, indicating in each case the strong and weak points and potentialities of future development.
  8. Comment on the following passage: “General low wages never caused any country to undersell its rivals; nor did general high wages ever hinder it from doing so.”
  9. Is there any inconsistency between the views that the foreign exchanges are determined (a) by the demand for and supply of bills of exchange, (b) by the relative purchasing power of the currencies of the countries concerned? What do you consider the correct view of the matter?
  10. “It is impossible for any country to finance a great war without an inflation of the currency.” Discuss this statement.

 

Monday, May 30, 1921. 1½–4½
RECENT ECONOMIC AND GENERAL HISTORY OF THE UNITED KINGDOM AND THE BRITISH EMPIRE.

  1. Show how Adam Smith’s Wealth of Nations influenced the economic policy of England. What influence had it outside England?
  2. Outline the different measures which have been taken since 1783 for reducing the burden of the National Debt.
  3. What were the main objects which Mr. Gladstone had in view, when selecting the sources from which his revenue was to be derived?
  4. What light is thrown by railway and banking history on the attitude of Parliament towards monopoly?
  5. How far was the distress of the hand-loom weavers and frame-work knitters in the middle of the nineteenth century caused by the introduction of steam power?
  6. What happened to the surplus agricultural population which before the Poor Law of 1834 was maintained, wholly or in part, at the expense of the parish?
  7. (a) “Down to 1870 England’s colonial policy was a policy of drift.”
    (b) “The theorists of 1830 were animated by a mastering passion for liberty, and it is because they succeeded in making it the basic principle of England’s colonial policy that the British Empire is to-day a fact of such cardinal importance in the world.”
    Which seems to you the more correct view, and why?
  8. “The discovery of gold precipitated Australia into manhood.”
    Discuss this; and point out the social problems confronting a new country which possesses precious metals in abundance.
  9. Describe the chief forms of land tenure existing in India, and account for their diversity.
  10. Examine the methods which have been adopted in different parts of the British Empire for the avoidance and settlement of industrial disputes.

 

Tuesday, May 31, 1921. 9–12.
SUBJECTS FOR AN ESSAY.

  1. “Une extrême justice est souvent une injure.”
  2. “In all essentials the problem of industrial peace resembles that of international peace.”
  3.  “The economic relationship of nations is essentially one of cooperation, rather than of rivalry: the prosperity of each country is to the advantage of the others.”
  4. “The nineteenth century was the outcome of French ideas and English technique.”
  5. The Restoration of the Gold Standard.

“From this unrest, so, early wrecked,
A future staggers crazy,
Ophelia of the Ages, decked
With woeful weed and daisy.”

 

Tuesday, May 31, 1921. 1½–4½.
RECENT ECONOMIC AND GENERAL HISTORY OF EUROPE AND THE UNITED STATES.

  1. Contrast the map of Europe, as it was in 1815, with the map of Europe as it is to-day.
  2. To what causes do you attribute the instability of the political institutions of France between 1789 and 1870?
  3. “The German Empire was built more truly on coal and iron than on blood and iron.”
    What evidence is there to support this statement?
  4. “Le Gouvernement provisoire de la République française s’engage à garantir l’existence de l’ouvrier par le travail;
    Il s’engage à garantir du travail à tous les citoyens.” (Moniteur du 26 fevrier 1848.)
    What was the origin of this decree, and to what did it lead?
  5. “Gewiss ist es der Kaiser gewesen und konnte nur der Kaiser sein, der den Fürsten schliesslich entlassen hat, aber die moralische Autorität des Mannes, der das deutsche Reich geschaffen und 27 Jahre an der Spitze der Regierung gestanden hatte, war so ungeheuer, dass es für den Kaiser, der noch so wenig Regierungserfahrung hatte, eine moralische Unmöglichkeit gewesen wäre, sich von ihm zu trennen, wenn nicht eben der Kanzler durch sein Verhältniss zur Majorität des Reichstags sich in eine unhaltbare Position gebracht hätte.”
    Do you agree with this explanation of Bismarck’s dismissal? Give reasons for your answer.
  6. “In France the average unit of agriculture remains as small as ever it was, and its typical manager is still the working peasant or the very small farmer.”
    Why is it that the small farmer has held his own so successfully in France?
  7. Outline the main stages in the growth of the German Zollverein.
  8. Compare the railway systems of Germany and the United States, with particular regard to
    (a) the purposes for which they were planned;
    (b) the part played by the State in their development and control.
  9. How was it that the United States, which Huskisson regarded as our most serious shipping rival, had so small a mercantile marine in the second half of the nineteenth century?
  10. Why were the attractions of Canada, as a field for immigration, inferior to those of the United States during the greater part of the nineteenth century?
  11. What were the chief issues between North and South which led to the American Civil War?

 

Wednesday, June 1, 1921. 9–12.
GENERAL ECONOMICS. II.

  1. “If we wish to get a clear view of English industrial inefficiency at the present time we have only to contrast the average output per worker in American industrial establishments with the average output in English establishments; or to set side by side the average output of each cotton spindle employed in Japan with the corresponding figure for English spindles. In both comparisons England’s position is unsatisfactory almost beyond belief.” Criticise this argument.
  2. What meanings can be attached to the statement that a country is “over-populated”? In what sense, if any, can the statement be applied to this country at the present day?
  3. “The socialistic activities of modern states, by checking the natural play of those forces that tend to bring about the elimination of the relatively unfit, threaten a progressive deterioration of the qualities of the race.” Discuss this statement.
  4. Indicate the chief difficulties in the way of estimating accurately the change in the real earnings of any group of wage-earners in 1921, as compared with 1914.
  5. Are trade unions well advised (a) in attempting to prohibit the entry of female workers into specific occupations, (b) in insisting on equal pay for male and female workers in occupations where both are normally employed?
  6. “Plasticity of wage rates would be the sovereign remedy for the evil of unemployment.” Discuss this view.
  7. Analyse the various causes, both temporary and relatively permanent, which may bring about a disparity between a country’s recorded exports and imports. Illustrate from actual instances.
  8. Discuss the effects of increasing supplies of gold upon the short period and the long period rate of interest in a gold-standard country.
  9. “It is not an essential part of the principle of a gold standard that gold coins should freely circulate within the community adopting that principle, or even that gold coins should be freely minted.” Discuss this statement.
  10. “If all citizens were intelligently alive to their own interests and could also be depended on to be strictly honourable in all their dealings with the State, no taxes on commodities would be tolerated.” Comment.

 

Wednesday, June 1, 1921. 1½–4½.
EXISTING POLITICAL INSTITUTIONS.

  1. “Distrust of the elected legislature is the outstanding feature of the new German constitution.” Discuss this statement.
  2. Compare the functions and the effective powers of the Senate of the United States with those of any one other existing Senate.
  3. Indicate the limitations under which the Referendum is usually instituted, giving reasons for the precautions adopted.
  4. Contrast, in broad outline, the stages by which responsible government has been developed in the Dominions with the steps taken towards its establishment in India.
  5. Describe the relations of the Executive and the Legislature in modern Switzerland. What are their chief advantages and drawbacks? How far could the same system be made applicable in England?
  6. “British and American constitutionalism especially safe- guards the independence of the judiciary in order to ensure the liberty of the subject; European practice aims rather at rendering the powers of the administrators of the law effective against the individual with a view to the safety of the State.” Indicate the extent to which this may be considered a correct statement.
  7. “In America the President and his Cabinet possess executive and administrative functions only; the British Cabinet’s essential functions are of a widely different character.” Comment.
  8. “A written constitution is not the same as a rigid constitution.” Explain this dictum, illustrating from existing constitutions.
  9. Compare Canada with either Australia or the United States as a type of federal state.
  10. In what various ways are the powers of local authorities limited and controlled in the United Kingdom?
  11. “In a unitary state it is impossible to maintain two co-equal legislative chambers.” Comment on this.

 

Thursday, June 2, 1921. 9–12.
GENERAL ECONOMICS. III.

  1. “Eine Grubenabgabe keine Rente ist, mag sie auch oft so genannt werden. Denn ausgenommen den Fall, dass Gruben, Steinbrüche und so weiter praktisch unerschöpflich sind, muss der Überschuss ihres Ertrages über die direkten Auslagen zum Teil wenigstens als der Preis aus dem Verkaufe aufgespeicherter Güter angesehen werden.” Comment on this statement.
  2. “C’est un fait remarquable dans le cas de l’or, que d’un côté les prix des marchandises ayant augmenté depuis 1914 à cause de la diminution sur le marché et pour des autres raisons, la puissance d’achat de l’or à l’égard de ces marchandises a diminué et que d’un autre côté, l’or fait prime parce que de nombreux pays—spécialement les pays d’Asie—montrent une préférence marquée en sa faveur.” Indicate the phenomena thus summarised and investigate the conclusions arrived at.
  3. “It is often said that a high rate of interest hampers production and makes it dearer. This is false.” Examine this view.
  4. Is it to the advantage of a country like Great Britain that its people should invest abroad a large proportion of their annual savings? Discuss the probable economic consequences of attempts to restrict the export of capital.
  5. Examine the effect upon international trade of the payment of a large indemnity by one country to other countries. Is it the case that Great Britain could only receive an indemnity from Germany by increasing her imports of German goods to a corresponding extent?
  6. “A vast unfunded debt is a standing menace to the stability of the banking and currency position.” Discuss.
  7. What data would you require in order to form an opinion as to whether the inhabitants of any European country were under-taxed or over-taxed in comparison with those of other countries?
  8. “Monopolistic combinations promise both steadier levels of prices and steadier conditions for the wage earner than can be expected under the present system of anarchic competition.”
    Discuss this opinion.
  9. “There can be no rise in the value of labour without a fall of profits.” Ricardo. Critically examine this theory.
  10. Discuss the economic significance and consequences of modern advertising.
  11. Comment on the following: “The art of making yourself rich in the ordinary sense is equally and necessarily the art of keeping your neighbour poor.” Ruskin.

 

PART II

Monday, May 30, 1921. 9–12.
ECONOMIC PRINCIPLES.

  1. Write short notes on (a) elasticity of demand and supply, (b) consumer’s surplus, (c) economic rent, (d) quasi-rent.
  2. What determines the selling price of an article under conditions of (a) free competition of purchasers and producers, (b) monopoly?
  3. Examine the relation between the rate of interest and the supply of capital in various classes of investment.
  4. Define “normal profits,” and explain the part played by this conception in Marshall’s theory of distribution.
  5. Consider the advantages of fixing wages on a sliding scale based (a) on the cost of living, (b) on the selling price of the product of the industry, (c) on the aggregate output of the industry. Give instances of each method.
  6. Consider the arguments for and against a countervailing import duty on goods imported from a country, of which the currency is greatly depreciated in terms of the currency of the importing country.
  7. Would it be beneficial if a State Bank, having the right to issue inconvertible notes, were to make loans for all approved purposes without interest, provided the borrower was able to furnish war loan or similar stock with a 10 per cent, margin as security? Frame your answer so as to persuade a person inclined to hold the opposite view to your own.
  8. What would be your test of over-population? Apply it in a general way to present conditions in India, Germany, England and the United States.
  9. “On peut trouver entre les faits d’ordre économique des relations de dépendance que l’on peut essayer d’exprimer par des formules algébriques, alors même qu’on ne pourrait pas les traduire en chiffres.”
    Give three or four examples of this.
  10. “Kapital ist der Teil des Vermögens, welcher, selbst Produkt menschlicher Arbeit, wieder zur Produktion bestimmt ist. Nach dem Sprachgebrauche liegt sowohl der Begriff des Vorrats für künftige Nutzung, wie der des Erwerbsmittels darin doch erscheint es richtiger, das Kapital im Gegensatz zur Natur als bestimmten Teil des Vermögens hinzustellen und den Nachdruck auf die Art der Verwendung zu legen, welche die Güter finden sollen, weil sich so die schärfste Abgrenzung des Begriffs durchführen lässt. Es sind nicht darunter zu begreifen die Güter, deren Wert nicht geschätzt werden kann, wie persönliche Fähigkeiten, der Staat usw. und die freien Güter.”
    Restate the salient points of this definition in your words, and point out the respects in which a different view has been or might be taken.

 

Monday, May 30, 1921. 1.30–4.30.
THE ECONOMIC FUNCTIONS OF GOVERNMENT.

  1. “Now that Public Finance is deliberately used as an engine for the redistribution of wealth, the conception of justice in taxation has become meaningless.” Examine this view.
  2. On what principles should the cost of elementary education be distributed between the National Exchequer and the Local Authority concerned?
  3. Consider the case for the State endowment of motherhood.
  4. “Taxes on business transactions are open to all the same objections as taxes on commodities, and should be entirely abolished.” Consider this contention.
  5. Investigate the view that even in ordinary times the State should control the price and ration the supplies of essential articles of food.
  6. How far can the allocation of public funds be made in such a way that the marginal pound will yield equal advantage in each direction of public expenditure?
  7. Discuss with illustrations the obligation which rests on a modern State to take steps to conserve the wasting natural resources contained within its borders.
  8. “A municipality should not undertake to supply any article or service unless it is prepared to become the sole supplier.” Do you consider that this advice is sound?
  9. “In einer Zeit, in der man eifrig nach neuen Steuereinnahmen Umschau zu halten hat, ist es begreiflich, dass insbesondere angesichts der Umwerfungen der Werte in rasch aufblühenden Gemeinwesen nicht nur bei radikalen Bodenreformern, die zunächst eine Verstaatlichung des Bodens, eine Wertzuwachsteuer immerhin als Abschlagszahlung wünschten, sondern über diese Kreise hinaus der Ruf ertönte nach einer Steuer die 1. mühelos erworbene Gewinne für die Gesamtheit möglichst stark mit Beschlag belegen sollte, 2. auf die Benützer des Bodens nicht überwalzbar erschien.” Discuss.
  10. “Entre les trois procédés de couvrir les dépenses de la guerre par l’impôt, par l’emprunt, ou par l’association des deux, M. Helfferich est demeuré bien en arrière de ses collègues d’Angleterre et même de Russie. L’Angleterre a couvert par les recettes de l’impôt 25% des énormes dépenses de guerre.”
    Consider the arguments for the various policies pursued by the belligerents in this respect.

 

Tuesday, May 31, 1921. 9–12.
STRUCTURE AND PROBLEMS OF MODERN INDUSTRY.

  1. “The liner she’s a lady.”
    “The black Bilbao tramp.”
    Explain the problem of ocean freights, as between these two services.
  2. Discuss the advantages of the holding company and the merger respectively as methods of combination, (a) with reference to industrial efficiency, (b) from the financial standpoint.
  3. Consider the problem of “Key industries,” (a) as to definition, (b) as to a right policy of development.
  4. Describe the prevailing systems of land tenure in England. What is there to be said in favour of the landlord?
  5. “Unfair methods of competition are hereby declared un- lawful.” (Federal Trade Commission Act, Sect. 5.)
    By what principles should a Court be guided in enforcing this clause? Illustrate by reference to special practices.
  6. What features of the existing structure of industry lead to proposals for the limitation of profits ? Give instances of such proposals, and discuss their effects.
  7. Suppose a scheme to be established for the benefit of operatives, e.g. in the cotton or in the engineering industry; how would you define the “industry” for this purpose? Show by examples the problems which would arise.
  8. Compare the organisation of the Cotton and the Wool Textile Industries as regards (a) the market for the raw material, and (b) the market for the finished articles.
  9. On what grounds has the present organization of the British coal industry been criticised from the standpoint of efficiency?

 

Tuesday, May 31, 1921. 1.30–4.30.
DISTRIBUTION AND LABOUR.

  1. Examine the probable effect on the real incomes of house-owners if the cost of building increases, other prices generally remaining unchanged, (a) when there is no legal restriction on rent, (b) under such an act as is now in force.
  2. Suppose a gradual transference in income, owing to rising prices and wages, from owners of securities bearing fixed rates of interest to workmen. How is the rate of saving affected, and what are likely to be the ultimate effects on national income if the transference results in a higher standard of living for workmen?
  3. Discuss the incidence of compulsory provision of insurance against (a) unemployment, (b) sickness, if the contributions are paid in part by the state, in part by employers and in part by wage-earners.
  4. Examine the conditions under which the establishment of a minimum wage in an industry does not result in diminution of employment, illustrating your answer from the experience of Australia or of the Trade Boards Acts in the United Kingdom.
  5. Among the effects of the wage-changes in 1915 to 1920 were the diminution of inequalities of wages between industries, between occupations and between localities. How far were such inequalities due to economic causes and how far to custom or to accident?
  6. A workman’s needs generally increase between the ages of 20 and 40, while in many occupations wages are irrespective of age. Can any methods be suggested by which this anomaly can be avoided?
  7. The causes of poverty can be roughly classified as economic or personal. Indicate some methods by which trade organisation can remove the former and state organisation the latter.
  8. Analyse the difference between the methods of settling wage disputes by compulsory arbitration, voluntary arbitration, and such organisations as Whitley Councils, giving illustrations from the practice of this or any other country.
  9. During the generation before the war there is evidence of a steady increase of the number of salaried persons relatively to the number of wage-earners. Consider the effects of this tendency on the statistics of the growth of the national income as they are usually classified.

 

Wednesday, June 1, 1921. 9–12.
MONEY, CREDIT, AND PRICES.

  1. Write a brief essay on the Quantity Theory of Money, describing in outline at least two methods of approach, and stating which you yourself prefer.
  2. State the chief theoretical difficulties in compiling an index number of the cost of living which shall be valid for comparisons between different places over a considerable period of time. Is there any important theoretical distinction between “cost of living” index numbers and index numbers of the value of money generally?
  3. Analyse and compare the different ways in which the term inflation is used by the wise and the foolish.
  4. How do you explain the fall of prices during the past twelve months? Do you expect it to continue?
  5. Compare the position of the U.S. Federal Reserve Board in the banking system of the U.S. with that of the Bank of England in the British system.
  6. Give an account of the probable career from start to finish of a cotton bill drawn to finance the purchase of American cotton by a Liverpool merchant.
  7. Give a careful account of the chief factors which influence the foreign exchanges, and forecast, with your reasons, the probable level of the dollar, franc and mark exchanges six months hence.
  8. Restate the theory of international trade for the case of a set of countries of which the currencies are not on a gold basis but consist of unrelated inconvertible paper moneys.
  9. Make practical proposals for obtaining from Germany as large as possible a sum in payment of Reparation.

 

Wednesday, June 1, 1921. 1.30–4.30.
SUBJECTS FOR AN ESSAY.

  1. The Money Motive.
  2. Business men in politics.
  3. Frontiers.
  4. Types of national leadership in Industry.
  5. Education and Industry.
  6. Bolshevism.

 

Thursday, June 2, 1921. 9–12.
MISCELLANEOUS ECONOMIC QUESTIONS.

  1. “Light is the sovereign antiseptic, and the best of all policemen.” Do you regard this maxim as an adequate guide for the State to follow in its dealings with industrial combinations?
  2. Consider how far the ordinary arguments against artificial interference with prices are relevant to arrangements for controlling the price and rationing the supplies of bank loans.
  3. “The great national Trade Unions, by their insistence on uniformity of conditions, are really hindering the workman in his attempt to better his lot and to secure a greater share in the government of industry.” Comment on this opinion.
  4. Examine the view that there is a danger of experiments in scientific management being carried further than the true interest of the nation demands.
  5. “The policy of giving subsidies to unemployed workmen necessarily involves inflation of currency, and therefore aggravates the evils it is designed to cure.” Discuss this statement.
  6. In what ways, if any, could the State usefully intervene to check the wastes caused by competitive advertisement?
  7. Examine the arguments for compelling railway companies to carry workmen going to and from their work at unremunerative rates.
  8. Consider the proposal that wage agreements negotiated by bodies representing the majority of employers and employed in any trade should be made legally binding on the remainder of the trade.
  9. Can you suggest any practicable scheme for the issue of an international currency by the League of Nations, in order to facilitate the resumption of international trade?

 

Thursday, June 2, 1921. 1.30–4.30.
THE THEORY OF STATISTICS.

  1. “The precision of the average of the measurements of n objects selected at random from a large group is proportional to \sqrt{n}.” Explain carefully the meaning of this statement, discuss the conditions under which it is true, and give examples of its use.
    What modification is needed if n/N is not negligible, where N is the number of objects from which selection is made?
  2. How does the analysis, which gives the ordinate of the normal curve of error as the limit of nCpn+xppn+xqqn-x when n is great, break down if pn is not great, and what is the resulting form?
  3. Distribution according to age of railway guards.

Age

Number of persons per 1000

Under 15 years

0

15—

8
20—

48

25—

252

35—

334
45—

223

55—

125
65—

10

75 and over

0

1000

Estimate by any method of interpolation the relative numbers in the age groups 30 to 35 and 35 to 40 years. State the hypotheses on which the method rests.

  1. For the frequency group in question 3 calculate the average, median and mode, and obtain one measurement of deviation and of asymmetry.
    Give a verbal description of the group, using your results, in language which avoids technicalities as far as possible.
  2. A six-faced die is thrown seven times; the numbers of the pips shown in the first five throws are added (x) and also of those shown in the last five (y). Show that if the experiment is repeated the value of the product-sum coefficient of correlation between x and y tends to be 3/5.
    Discuss the question whether any useful meaning can be given to the coefficient of correlation (obtained by the product-sum formula) in a case where nothing is known of the genesis of x and y and their distribution is not normal.
  3. Describe any one method by which association between non-measurable attributes (such as intellectual ability of pairs of brothers) can be tested, and consider whether a valid numerical measurement of association can be obtained.
  4. Find (by any method) the equation of a regression plane applicable to the following table, regarding each of the 15 entries as of equal importance.
    Compute the corresponding values from the equation and comment on the differences between the observed and computed numbers.
    What further information would be needed to calculate regression coefficients’?

    Age of wife at marriage

    Children born per 100 couples by age of husband
    15 – 25 25 – 35

    35 – 45

    15–20 years

    813 714 629
    20–25 661 595

    540

    25–30

    498 462 434
    30–35 364 341

    322

    35–40

    171 160

    150

  5. In a certain population the number of males over 25 years old is 875712, and the numbers who were 25, 26, 27, 28, 29 years last birthday respectively 29556, 29336, 29078, 28816, 28542. From these data, fill in the following, where the letters have the meanings usual in life tables:

    Age

    lx Lx dx qx

    mx

    24

    26

    27

    28

    29

If qx is the rate of mortality, mthe central death rate, and \mu_{x} the force of mortality, show that q_{x}= \frac{2m_{x}}{2+m_{x}} and approximately \mu_{x}= m_{x-\frac{1}{2}}.

  1. What data are necessary for measuring how far a low birth-rate is compensated by low infant and child mortality? Show in some detail the form of statistical results obtainable.

 

Thursday, June 2, 1921. 1.30–4.30.
ECONOMIC CONDITIONS IN ENGLAND 1823-1828,
AND CONTEMPORARY SOCIAL THOUGHT.

  1. Give an account of Place’s efforts to defeat the reactionary intentions of the Government in 1825 with regard to the Combination Act of 1824.
    How far is he justified in his statement that “Ultimately the Act (of 1825) differed very little from Mr Hume’s Act. It is substantially the same”?
  2. On what grounds did Huskisson advocate
    (a) the retention of the “long haul” clause in the Navigation Act,
    (b) imperial preference?
    Was the U.S.A. justified in regarding the latter as a continuance of the old tariff discriminations?
  3. “The question, then, looking at it practically, is this: In what degree is Prohibition better, as against smuggling, than a well-regulated duty?—by which I mean, a duty sufficient to protect the British manufacturer, without being so high as to afford a premium to the smuggler.” (Huskisson: Speech on the Silk Manufacture, Feb. 1826.)
    Show how this consideration guided Huskisson in the changes which he made in the customs tariff.
  4. What were the leading ideas in the programme of the early London Cooperators, and from what source or sources were they derived?
  5. Why were the problems of the Irish Poor and the English Hand-loom Weavers treated by the Emigration Committees of 1826 and 1827 as a single problem?
  6. Show how the fear of a redundant population dominated the recommendations of the Emigration Committees of 1826 and 1827.
  7. “In Lancashire there appear to be among the Hand-loom Weavers two classes almost wholly distinct from each other; the one who, though they take in work in their own homes or cottages, are congregated in the large manufacturing towns; and the other, scattered in small hamlets or single houses in various directions throughout the manufacturing country.” (Emigration Committee, 1827.)
    Was this distinction of long standing? Was it peculiar to Lancashire?
  8. Illustrate from the Rural Rides Cobbett’s first-hand acquaintance with agricultural conditions.
  9. “The Ricardian Socialists, though fairly unanimous in their criticisms of the existing system, differed greatly as to remedies.”
    Illustrate this from the writings of William Thompson, Thomas Hodgskin and J. F. Bray.
  10. If you were engaged in an investigation into social conditions in England (1823-8), which are the trades about which you would find it most difficult to obtain information, and why?

 

Thursday, June 2, 1921. 1.30–4.30.
INTERNATIONAL LAW.

  1. Describe the system of Colonial Mandates in the Covenant of the League of Nations. Indicate what are the important changes introduced and, in particular, consider the question of the sovereignty over the mandated areas.
  2. How far do you consider that the principle of “Self- Determination” has a basis in International Law?
    Comment on the following statement of Mr Lansing: “It is an evil thing to permit the principle (i.e. of Self-Determination) to continue to have the apparent sanction of the nations when it has been thoroughly discredited.”
  3. Compare the position in International Law of the Suez and Panama Canals.
  4. Examine the following statement of Lord Stowell: “In my opinion there exists a general rule in the maritime jurisprudence of this country by which all trading with the public enemy, unless with the permission of the sovereign, is interdicted. It is not a principle peculiar to the maritime law of this country.”
    On what grounds is such a principle to be justified?
  5. Comment on one of the following passages:
    (a) “II y a donc, voulue par la conscience universelle, une certaine limitation aux moyens d’hostilités, limitation encore bien imparfaite et qu’il est ambitieux de décorer du nom pompeux de lois de la guerre. Disons, plus modestement, qu’il existe des usages de la guerre dont le droit international demeure le guide moral et l’inspirateur.”
    (b) “Gegenüber der rechtmässigen Ausübung des Anhaltungs-, Durchsuchungs- und Wegnahmerechts gibt es für das Kauffahrteischiff kein Recht der Notwehr. Die Notwehr ist begrifflich die Verteidigung gegenüber einem rechtswidrigen Eingriff in ein Rechtsgut. Dies gilt allgemein für die neutralen wie für die feindlichen Kauffahrteischiffe. Die letzteren nehmen keine Ausnahmestellung ein. Auch sie besitzen kein Recht der Notwehr.”
  6. To what treatment are merchant ships belonging to the citizens of a belligerent state which are in the ports of the other belligerent at the outbreak of war subjected by International Law?
    What course was taken by Great Britain, with regard to German and Austrian ships in British ports at the outbreak of the war in 1914?
  7. Discuss the origin and development of the doctrine of “Continuous Voyage.”
  8. Do you consider that any violation of neutrality has been committed in any of the following cases?
    (a) The export of munitions by neutral manufacturers to one of two belligerents.
    (b) The granting permission by a neutral state for the passage through its territory of the sick and wounded of a belligerent’s army.
    (c) The granting of permission by a neutral state to belligerent warships to effect repairs in its ports.
  9. Discuss the propriety of the use of the term “Blockade” in relation to the measures taken by Great Britain under the Order in Council of 11 March 1915.
  10. What is the effect of lapse of time upon the binding force of a treaty?
    Discuss the action of Austria in annexing Bosnia and Herzegovina in 1908.

 

Friday, June 3, 1921. 9–12.
POLITICAL SCIENCE.

  1. “By Liberty I mean the assurance that every man shall be protected in what he believes his duty, against the influence of authority and majority, custom and opinion.” (Acton.)
    Use this opinion to show how you define Liberty.
  2. How has the argument regarding the rights of property been affected by the development of modern industrial conditions?
  3. State and examine Rousseau’s distinction between the General Will and the Will of All.
  4. Discuss Mill’s view of the conditions essential to the success of representative government.
  5. What are the conditions or grounds of objection which give a minority the strongest right to refuse obedience?
  6. On what principle should the relations between a first and a second chamber be regulated in a modern democracy?
  7. Give a critical estimate of the position which has developed between the State and Labour, through the Social Reform movement of the last generation in England.
  8. Explain the alternative implied in Weltmacht oder Niedergang. What reactions on the life of the great State have been attributed to the development of colonial possessions?
  9. What are the main considerations which determine a sound theory of punishment?
  10. “The separation of powers in politics corresponds to the fixity of species in natural science; and both ignore evolution.”
    Comment on this statement.

Source: Cambridge University Economics Tripos Papers, 1921-1926. With the papers set in the Qualifying examinations 1925 & 1926. Cambridge at the university press 1927, pp. 1-21.

Image Source: Kings College, Cambridge England. Library of Congress Prints and Photographs Division, Washington, D.C. 20540.

Categories
Exam Questions M.I.T. Problem Sets Syllabus Undergraduate

M.I.T. Principles of Microeconomics, course materials. 1994-2005

 

Today’s post takes Economics in the Rear-view Mirror on a short journey to the very recent past.  Instead of transcribing archival material and publications from the period 1870-1970, I thought I would see what trawling the 341 billion web pages in the internet archive, Wayback Machine, might yield us.

On Christmas eve of 1996 Wayback Machine first captured webpages for the principles of microeconomics course taught to undergraduates at M.I.T. (14.01). Below you will find links to the archived lecture plans, problem sets and questions/answers for midterm and final examinations that I have been able to find. Spoiler alert: there are gaps in this archival record, but still one finds plenty of useful items, now more conveniently ordered. 

But first I share a few paragraphs from my paper “Syllabi and Examinations” that suggest the method in my madness. 

 

_________________

On the virtual informational frontier in the history of economics

…historians of recent economics are facing information-engineering challenges of learning to harness the power from the enormous current of weblog postings, tweets, working papers, media transcripts and exploding data bases to study the processes of scientific innovation and diffusion.  The pedagogy of walk-talk-and-chalk has almost become relegated to the stuff of legend, and successive waves of duplication technologies have been forced to yield to the “pdf-ing” of lecture notes, syllabi, spreadsheets, and problem sets. Video and audio recordings of lectures, panel discussions, and interviews also contribute to a genuine curse of dimensionality confronting historians of contemporary economics.

Now we can imagine a virtual divide in our informational past that marks a frontier between the methodological problems associated with the relative scarcity of written artifacts relevant for the study of the earlier evolution of the education and training of economists and the current problems of judiciously sampling from an ever expanding big data universe.  But whether as historians we are working one side of this frontier or the other, it makes great sense to embed our specific empirical concerns within a common framework, assuming a great arc of continuity (nobody said smooth!) that connects 1918 with, say, 2018 with respect to the scope and methods of economics. Without a common framework, our respective narratives would resemble tunnel building from opposite sides of a mountain with the most likely result being two noncommunicating parallel tunnels in the end. Does anyone really think there is a parallel Harvard, Chicago, Columbia, Wisconsin, Michigan universe? Of course not, we really did get here from there.

 

Source:  Irwin L. Collier. Syllabi and Examinations in History of Political Economy, Vol. 50, No. 3 (September 2018), pp. 587-595.

_________________

MIT 14.01
PRINCIPLES OF MICROCONOMICS

Spring 1994
Professor Jeffrey Harris

Final Exam and Alternate Final Exam (questions)

Fall 1994
Professor Franklin Fisher

Midterm Exam 1 (questions and answers)

Final Exam & Alternate Final Exam (questions)

Spring 1995
Professor Jeffrey Harris

Problem Sets with Solutions

Midterm Exam 1 (questions and answers)

Midterm Exam 2 (questions and answers)

Final and Conflict Final Exams (questions)

Fall 1995
Professor Franklin Fisher

Problem Sets with Solutions

Midterm Exam 1 (questions and answers)

Final and Alternate Final Exams (questions)

Spring 1996
Professor Jeffrey Harris

Problem Sets with Solutions

Midterm Exam 1 (questions and answers)

Midterm Exam 2 (questions and answers)

Final and Conflict Exam (questions)

Fall 1996
Professor Jeffrey Harris

Textbook:  Earl L. Grinols, Microeconomics (Boston: Houghton-Mifflin, 1994). “The textbook differs from that assigned in recent past semesters.”

Course home page

Syllabus

Additional Course Information

Schedule

Problem sets and Solutions

Midterm 1 (with answers)

Midterm 2 (questions and answers)

Midterm 2, alternate (questions and answers)

Final and Conflict Exams (questions)

Spring 1997

No Wayback Machine captures found…yet!

Fall 1997
Professor Jeffrey Harris

Probable Textbook: Earl L. Grinols, Microeconomics (Boston: Houghton-Mifflin, 1994).

Course home page

[For some reason all the links go back to Fall 1996]

Spring 1998

No Wayback Machine captures found…yet!

Fall 1998
Professor Jeffrey Harris

Textbook: Earl L. Grinols, Microeconomics (Boston: Houghton-Mifflin, 1994).

Course home page

Syllabus

Schedule

Spring 1999
Professor Jonathan Gruber

Textbook: Jeffrey M. Perloff, Microeconomics (Addison Wesley Longman, 1999).

Course home page

Syllabus

Fall 1999
Professor Jeffrey Harris

Textbook: Jeffrey M. Perloff, Microeconomics (Addison Wesley Longman, 1999).

Syllabus

Spring 2000
Professor Jonathan Gruber

Textbook: Jeffrey M. Perloff, Microeconomics (Addison Wesley Longman, 1999).

Syllabus

Schedule

Fall 2000
Professor Jonathan Gruber

Textbook: Perloff, Jeffrey M. Microeconomics. 1st Edition. Addison-Wesley.

Course home page

Syllabus

Schedule

Spring 2001
Professor Christopher Snyder

Textbook: Pindyck and Rubinfeld, Microeconomics, 5th ed.

Course home page

Syllabus

Schedule

Fall 2001
Professor Jeffrey Harris

Textbook: Pindyck & Rubinfeld, Microeconomics, 5th Edition (Prentice Hall, 2001).

Course home page

Syllabus

Schedule

Midterm 1 with answers

Spring 2002
Professor Paul Joskow

Textbook: Pindyck & Rubinfeld, Microeconomics, 5th Edition (Prentice Hall, 2001).

Course home page

Syllabus

Schedule

Fall 2002
Professor Jonathan Gruber

Textbook: Jeff Perloff, Microeconomics, 2nd Edition (Addison Wesley Longman, 2001).

Course home page

Syllabus

Schedule

Midterm 1 with answers

Spring 2003
Professor Paul Joskow

Textbook: Pindyck and Rubinfeld, Microeconomics, 5th Edition.

Course home page

Syllabus

Schedule

Fall 2003
Professor Jonathan Gruber

Textbook: Jeffrey M. Perloff, Microeconomics, 3rd Edition.

Course home page

Syllabus

Schedule

Midterm 1 (Solutions)

Midterm 2 (Solutions)

Spring 2004
Professor Paul Joskow

Textbook: Pindyck and Rubinfeld, Microeconomics, 5th Edition.

Course home page

Syllabus

Schedule

Fall 2004
Professor Jonathan Gruber

Textbook: Jeffrey M. Perloff , Microeconomics, 3rd Edition.

Course home page

Syllabus

Midterm 1 (questions)

Spring 2005
Professor Jeffrey Harris

Textbook: Microeconomics, Robert S. Pindyck, Daniel L. Rubinfield, Prentice Hall, June 30, 2004 (6th edition).

Course home page and syllabus

Schedule

 

Image:  Mr. Peabody (dog) and Sherman (boy) activating the original WABAC Machine.

Categories
Exam Questions Harvard Leontief Undergraduate

Harvard. Undergraduate mathematical economics. Schumpeter, Leontief, Goodwin. 1933-1950

 

 

Joseph Schumpeter introduced a one semester undergraduate course “Introduction to the Mathematical Treatment of Economic Theory” in the first semester of the 1933-34 academic year at Harvard. Schumpeter taught the course three times and it was taught from 1935-36 through 1947-48 by Wassily Leontief. The course was then continued by Richard Goodwin in 1949-50. This post presents a grab-bag of information that includes early and a late course description, annual enrollment data, a course outline from 1945-46 and five exams. Links to all earlier posts for the course available at Economics in the Rear-view Mirror have been included as well.

Some of the backstory to this course is included in this earlier post (memo by Crum of 4 April 1933 and a list of topics to be covered).

_______________

Course Announcement, 1933-34

Economics 8a 1hfIntroduction to the Mathematical Treatment of Economic Theory

Half-course (first half-year). Mon., 4 to 6, and a third hour at the pleasure of the instructor. Professor Schumpeter, and other members of the Department.

Economics 8a is open to those who have passed Economics A, and Mathematics A, or its equivalent. The aim of this course is to acquaint such students as may wish it with the elements of the mathematical technique necessary to understand the simpler contributions to the mathematical theory of economics.

Source:  Announcement of the Courses of Instruction offered by the Faculty of Arts and Sciences 1933-34 (Second edition) in Official Register of Harvard University, Vol. XXX, No. 39 (September 20, 1933), p. 126.

*  *  *  *  *  *  *

Course Enrollment, 1933-34

[Economics] 8a 1hf. Professor Schumpeter. — Introduction to the Mathematical Treatment of Economic Theory.

15 Graduates, 3 Seniors, 5 Others. Total 23.

Source: Harvard University. Report of the President of Harvard College and Reports of Departments for 1933-1934, p. 85.

*  *  *  *  *  *  *

Exam not found for Economics 8a, 1933-34

_______________

Course Enrollment, 1934-35

[Economics] 8a 1hf. Professor Schumpeter. — Introduction to the Mathematical Treatment of Economic Theory.

2 Seniors, 1 Junior, 1 Sophomore. Total 4.

Source: Harvard University. Report of the President of Harvard College and Reports of Departments for 1934-1935, p. 81.

*  *  *  *  *  *  *

 1935 final exam questions.

_______________

Course Enrollment, 1935-36

[Economics] 8a 2hf. Asst. Professor Leontief. — Introduction to the Mathematical Treatment of Economic Theory.

4 Juniors, 2 Sophomores. Total 6.

Source: Harvard University. Report of the President of Harvard College and Reports of Departments for 1935-1936, p. 82.

*  *  *  *  *  *  *

Implicit course outline and course readings with the 1936 exam questions.

_______________

Course Enrollment, 1936-37

[Economics] 4a 2hf. Asst. Professor Leontief. — Introduction to the Mathematical Treatment of Economic Theory.

1 Graduate, 2 Seniors, 3 Juniors, 2 Sophomores, 1 Other. Total 9.

Source: Harvard University. Report of the President of Harvard College and Reports of Departments for 1936-1937, p. 92.

*  *  *  *  *  *  *

Final Examination, 1936-37
HARVARD UNIVERSITY
ECONOMICS 4a

Answer at least THREE questions: one in each group

Group I

  1. Discuss the relation between the production function of an enterprise and its cost curve.

 

Group II

  1. Given a cost of a single plant:
    C=\frac{1}{A+X}+BX
    where indicates the total cost, the total output, and the magnitudes of the two constants are such
    that A< 0 and B> 1/A.
    Derive the total cost curve of an enterprise which consists of two identical plants of this kind.
  2. A monopolist sells in two markets a commodity produced without costs. The total revenue, R1, obtained from the sale of qunits of this commodity in the first market is given by:
    {{R}_{1}}=A{{q}_{1}}+Bq_{1}^{2}\text{ }\left( A>0,\text{ }B<\text{ }0 \right)
    The sale of qunits in the second market nets:
    {{R}_{2}}=K{{q}_{2}}+Lq_{2}^{2}\text{ }\left( K>0,\text{ }L<\text{ }0 \right)
    Compute the prices which this monopolist would charge (a) with discrimination between the two markets; (b) without discrimination.

 

Group III

  1. Prove that marginal costs are increasing in the point of minimum average costs.
  2. Prove that a tax on profits cannot affect the output of an enterprise unless it induces it to suspend its operations.

 

Source: Harvard University Archives. Examination Papers. Finals 1937. (HUC 7000.28) Vol. 79. Faculty of Arts and Sciences. Papers Printed for Final Examinations. History, History of Religions, …, Economics, …, Military Science, Naval Science. January-June, 1937.

_______________

Course Enrollment, 1937-38

[Economics] 4a 2hf. Asst. Professor Leontief. — Introduction to the Mathematical Treatment of Economic Theory.

2 Graduates, 2 Seniors, 6 Juniors, 1 Sophomore. Total 11.

Source: Harvard University. Report of the President of Harvard College and Reports of Departments for 1937-1938, p. 85.

*  *  *  *  *  *  *

Final Examination, 1937-38
HARVARD UNIVERSITY
ECONOMICS 4a2

Answer THREE questions including question 1. Devote to discussion of question 1 about one hour and a half.

  1. Discuss fully the relation between the production function and the cost curve of an enterprise.
  2. Given:
    1. The cost curve of a monopolist:
      C= A+ BQ+ CQ2
      C indicates the total cost, the total output, A, B, C,are given constants.
    2. The demand function for his product in Market I.
      q1= a1b1p1
      qis the quantity consumed for his product in Market I at the price p1.
      a1and bare given constants
    3. The demand function for his product in Market II.
      q2= a2b2p2
      q2is the quantity taken in at the price p2;
      aand bare given constants.
      The monopolist is able to discriminate between the two markets provided the difference between the two prices is not larger than K
      Find (and express in terms of the given constants) that the value of Kwhich would maximize the sales qin the first market.
  3. Given:
    1. A, monopolist’s cost curve:
      C = A+ BQ+ CQ
    2. The demand curve for his product:
      p= a bQ
      stands for total costs, Q for total output, for the market price, A, B, C, d, and are constants.
      A subsidy at dollars is paid to the monopolist per unit of output.
      Find how large the subsidy must be in order to induce him to produce and sell twice as much as he would without the subsidy.
  4. Is it possible that the average costs of an enterprise are increasing with the output while the marginal costs are decreasing at the same time?
    Give and answer and demonstrate that it is correct.

 

Source: Harvard University Archives. Harvard University Final Examinations, 1853-2001. (HUC 7000.28) Box 4. Faculty of Arts and Sciences. Papers Printed for Final Examinations. History, History of Religions, …, Economics, …, Military Science, Naval Science. January-June, 1938.

_______________

Course Enrollment, 1938-39

[Economics] 4a 2hf. Asst. Professor Leontief. — Introduction to the Mathematical Treatment of Economic Theory.

2 Graduates, 2 Seniors, 2 Juniors, 1 Sophomore. Total 7.

Source: Harvard University. Report of the President of Harvard College and Reports of Departments for 1938-1939, pp. 97-98.

*  *  *  *  *  *  *

Exam not found for Economics 4a, 1938-39

_______________

Course Enrollment, 1939-40

[Economics] 4a 2hf. Associate Professor Leontief. — Introduction to the Mathematical Treatment of Economic Theory.

1 Graduate, 1 Sophomore. Total 5.

Source: Harvard University. Report of the President of Harvard College and Reports of Departments for 1939-1940, p. 98.

*  *  *  *  *  *  *

Final Examination, 1939-40
HARVARD UNIVERSITY
ECONOMICS 4a2

Answer four questions including question 1.

  1. Discuss the relation between the marginal costs of an enterprise and the marginal productivities of the factors used in production.
  2. An enterprise manufactures two commodities X and Y, using two factors of production, V and W. The production function is x(yb– 1) = vnwm.
    Given the prices px, py, pvand pwwrite down the equations which determine the most profitable outputs of X and Y and the corresponding inputs of V and W.
  3. Given:
    1. The total cost curve of a monopolist
      C = A + Kxand
    2. the market demand curve for his product
      p = B – Lx,
      p is the price and x the quantity of the commodity produced and sold. A, K, B and L are positive constants.
      An excise tax of z dollars per unit of output is being levied.
      What magnitude of z (expressed in terms of the given constants) would maximize the total tax receipts?
  4. Prove that the price of labor will exceed its marginal value productivity if
    1. labor is the only factor of production used in manufacture of a given commodity,
    2. the producer of this commodity sells his output on a purely competitive market, but is the only (“monopsonistic”) buyer of the particular kind of labor used in his plant,
    3. The supply curve of labor is negatively inclined.
  5. Discuss the problem of price discrimination by a monopolist.

 

Source: Harvard University Archives. Harvard University Final Examinations, 1853-2001. (HUC 7000.28) Box 5. Faculty of Arts and Sciences. Papers Printed for Final Examinations. History, History of Religions, …, Economics, …, Military Science, Naval Science. June, 1940.

_______________

Economics 4a not offered in 1940-41

_______________

Course Enrollment, 1941-42

[Economics] 4a 2hf. Associate Professor Leontief. — Introduction to the Mathematical Treatment of Economic Theory.

1 Graduate, 5 Seniors, 8 Juniors, 3 Sophomores, 1 Freshman. Total 18.

Source: Harvard University. Report of the President of Harvard College and Reports of Departments for 1941-1942, p. 62.

*  *  *  *  *  *  *

Course Outline Economics 4a 1941-42 (and 1942-43)

https://www.irwincollier.com/harvard-intro-to-mathematical-economics-schumpeter-leontief-1935-42/

*  *  *  *  *  *  *

Final Examination, 1941-42
HARVARD UNIVERSITY
ECONOMICS 4a

Answer one question in each of the following three groups:

(a) 1 or 2
(b) 3 or 4
(c) 5 or 6

  1. Describe in detail the relation between a production function and the corresponding cost function.
  2. Show that the slope of a supply curve of a single enterprise is positive.
  3. Show that a total cost curve can be of such a shape that the marginal costs are increasing but the average costs decreasing throughout its whole length. Give example.
  4. The cost curve of an enterprise is
    C = A + x + Bx2+ Kx3
    (C are the total costs, x – the output, A, B, and K – constants).
    What is the lowest competitive price at which the owner will find it profitable to operate the plant rather than to cease production entirely?
  5. An enterprise consists of two identical plants. Each has a following cost curve:
    C = A + Bx2+ x3
    (C are the total costs, x – the output, A and B are constants).
    Compute the combined cost curve of the whole enterprise.
  6. Given a production function y = f(x,z)
    (y is the amount of product, p– its price, x and z inputs of two factors, pand p– their respective prices.)
    The producer maximizes his profits under conditions of pure competition. Show that an increase of the price pof factor x will reduce the amount (x) of this factor used in the process of production.

 

Source: Harvard University Archives. Harvard University Final Examinations, 1853-2001. (HUC 7000.28) Box 6. Faculty of Arts and Sciences. Papers Printed for Final Examinations. History, History of Religions, …, Economics, …, Military Science, Naval Science. June, 1942.

_______________

Course Description, 1942-43

Economics 4a 1hfIntroduction to the Mathematical Treatment of Economic Theory. Half-course (first half-year). Mon.4 to 6. Associate Professor Leontief.

Economics A and Mathematics A, or their equivalents, are prerequisites for this course.
The course is intended to instruct beginners in economic theory (having had elementary mathematical training) in the application of elementary mathematical methods in economics and at the same time to enable them to understand some of the major contributions to economic theory made by such writers as Marshall, Cournot, Walras, and Edgeworth.

Source:  Official Register of Harvard University, Vol. XXXIX, No. 45 (June 30, 1942). Division of History, Government, and Economics Containing an Announcement for 1942-43. 

*  *  *  *  *  *  *

Course Enrollment, 1942-43

[Economics] 4a 1hf. Associate Professor Leontief. — Introduction to the Mathematical Treatment of Economic Theory.

1 Graduate, 2 Seniors, 4 Juniors, 2 Sophomores, 1 Public Administration. Total 10.

Source: Harvard University. Report of the President of Harvard College and Reports of Departments for 1942-1943, p. 46.

*  *  *  *  *  *  *

Exam not found for Economics 4a, 1942-43

_______________

Course Enrollment, 1943-44

[Economics] 4a. (winter term) Associate Professor Leontief. — Introduction to the Mathematical Treatment of Economic Theory.

2 Juniors in ROTC, 1 Radcliffe, 3 Seniors, 4 Navy (V-12). Total 10.

Source: Harvard University. Report of the President of Harvard College and Reports of Departments for 1943-1944, p. 56.

*  *  *  *  *  *  *

Exam not found for Economics 4a, 1943-44

_______________

Economics 4a not offered in 1944-45

_______________

Course Enrollment, 1945-46

[Economics] 4a. (fall term) Associate Professor Leontief. — Introduction to the Mathematical Treatment of Economic Theory.

1 Senior, 2 Juniors, 3 Sophomores, 2 Radcliffe. Total 8.

Source: Harvard University. Report of the President of Harvard College and Reports of Departments for 1945-1946, p. 58.

*  *  *  *  *  *  *

Course Outline, 1945-46

INTRODUCTION TO THE MATHEMATICAL TREATMENT OF ECONOMIC THEORY
Economics 4a
1945-46, Fall Term

  1. Introductory remarks.
    Profit function.
    Maximizing profits.
  2. Cost functions: Total costs, fixed costs, variable costs, average costs, marginal costs, increasing and decreasing marginal costs.
    Minimizing average total and average variable costs.
  3. Revenue function.
    Price and marginal revenue.
    Demand function
    Elasticity and flexibility.
  4. Maximizing the net revenue (profits).
    Monopolistic maximum.
    Competitive maximum.
    Supply function.
  5. Joint costs and accounting methods of cost imputation.
    Multiple plants.
    Price discrimination.
  6. Production function.
    Marginal productivity.
    Increasing and decreasing productivity.
    Homogeneous and non-homogeneous production functions.
  7. Maximizing net revenue, second method.
    Minimizing costs for a fixed output.
    Marginal costs and marginal productivity.
  8. Introduction into the theory of consumers’ behavior.
    Indifference curves and the utility function.
  9. Introduction to the theory of the market.
    Concept of market equilibrium.
    Duopoly, bilateral monopoly.
    Pure competition.
    Monopoly.
  10. Time lag and time sequences.
  11. Introduction into the theory of general equilibrium.

 

Reading: R. G. D. Allen, Mathematical Analysis for Economists.

Evans, Introduction into Mathematical Economics.

Antoine Cournot, Researches into the Mathematical Principles of the Theory of Wealth.

Jacob L. Mosak, General Equilibrium Theory in International Trade.

Weekly problems.

Source: Harvard University Archives. Syllabi, course outlines and reading lists in Economics, 1895-2003. HUC 8522.2.1, Box 3, Folders “1945-1946 (1 of 2)”.

*  *  *  *  *  *  *

Final Examination, 1945-46
1945-46
HARVARD UNIVERSITY
ECONOMICS 4a
Introduction to Mathematical Economics

Answer any three questions.

  1. Show the relationship between the total cost curve and the supply curve of an enterprise.
  2. Show that, at the point of optimum output, the marginal costs of an enterprise are equal to the price of any cost factor divided by its marginal productivity.
  3. A consumer has an income of qdollars in the first and of ydollars in the second year. Although the combined expenditures in the two years equal y1+ y2he can spend more than yin the first year, and correspondingly less in the second year or vice versa. In both years, he purchases one kind of consumers’ goods, its price being pdollars in the first and pdollars per unit in the second year. The utility function which the consumer maximizes is u= f(x1, x2) where is the utility level, xand xthe quantities consumed in the first and second year respectively.
    1. Derive the equations which determine the optimum magnitudes of xand x2.
    2. Show that an increase of the price p1, with p2, y1,yremaining constant, might increase x1.
  4. The demand, q, for the product of a monopolist depends upon the price, p, of his produce and the amount of money, y, which he spends on advertising. The total production cost, c, depends upon the quantity of output, q. Given the demand function: q=\frac{A}{p}+{{y}^{{1}/{4}\;}}-p
    and the total (production) cost function = q
    where is a positive constant;
    Determine the output, the price, and the advertising outlay which would maximize the profits (total revenue minus total outlay) of this enterprise.
  5. The well-being, u, of a worker depends upon the amount, x, of consumers’ goods which he can buy with his daily wage, and the number of hours of leisure, y, which remain to him after he finishes his daily work:
    u= f(x, y)

    1. Derive the equations determining the number of hours (call it l) of daily work which he will be willing to do at the wage of dollars per hour, if the price of the consumers’ good is dollars per unit.
    2. Show that an increase of the hourly wage rate might reduce the number of hours which the worker will choose to work.

 

Source: Harvard University Archives. Harvard University Final Examinations, 1853-2001. (HUC 7000.28) Box 11. Faculty of Arts and Sciences. Papers Printed for Final Examinations. History, History of Religions, …, Economics, …, Military Science, Naval Science. January, 1946.

_______________

Economics 4a not offered in 1946-47

_______________

Course Enrollment, 1947-48

[Economics] 4a. Professor Leontief. — Introduction to the Mathematical Treatment of Economic Theory (Sp).

2 Graduates, 6 Seniors, 8 Juniors, 1 Sophomore, 2 Public Administration, 1 Radcliffe. Total 20.

Source: Harvard University. Report of the President of Harvard College and Reports of Departments for 1947-1948, p. 89.

*  *  *  *  *  *  *

Reading list and midterm and final examination question, 1947-48

_______________

Economics 4a not offered in 1948-49

_______________

Course Enrollment, 1949-50

[Economics] 104 (formerly Economics 4a). Assistant Professor Goodwin. — Introduction to the Mathematical Treatment of Economic Theory (Sp).

3 Graduates, 6 Seniors, 1 Junior, 2 Sophomores, 1 Public Administration, 1 Radcliffe. Total 14.

Source: Harvard University. Report of the President of Harvard College and Reports of Departments for 1949-1950, p.72.

*  *  *  *  *  *  *

Course Texts on Library Reserve, 1945-46

R.G.D. Allen. Mathematical analysis for economists

W.L. Crum. Rudimentary mathematics for economists and statisticians

P.A. Samuelson. Foundations of economic analysis.

Source: Harvard University Archives. Syllabi, course outlines and reading lists in Economics, 1895-2003. HUC 8522.2.1, Box 4, Folders “1949-1950 (1 of 3)”.

_______________

Image Sources: Schumpeter and Leontief from Harvard Class Album 1950, Goodwin from Harvard Class Album 1951.

Categories
Austria Economists Exam Questions Johns Hopkins Methodology

Johns Hopkins. Final exam for Fritz Machlup’s methodology course, 1956

 

 

Besides the questions for the final exam in Fritz Machlup’s course on the methodology of economics from the first semester of the 1955-56 academic year at Johns Hopkins University, I include the following photo from the 1956 yearbook Hullabaloo (p. 15) that identifies neither the speaker nor the seminar. While this is about as generic a seminar photo as one can imagine, I have something more than a mere suspicion that we are looking at Fritz Machlup in action. Perhaps some visitor with a keener forensic eye can confirm or reject my tentative identification in a comparison of the above portrait of Machlup reading himself with the speaker in the mystery seminar. The third man on the right, counting from the speaker, sure looks like a young Evsey Domar.

My hunch is based on the following picture of almost certainly the same seminar room in 1963 from the Carl Christ memorial website at Johns Hopkins.

 

______________

THE JOHNS HOPKINS UNIVERSITY
METHODOLOGY
18.601

Professor Fritz Machlup
January 27, 1956

Answer five questions, one from each group.
Write on loose sheets of paper; start a new sheet for each question.
Identify each sheet by the Question Number in the left corner and your Examination Number (which you draw before the examination) in the right corner; your name should appear nowhere.
You are on your honor not to use notes or to give or accept advice.

I.

  1. According to Poincaré, “a priori propositions are irrefutable because they are really firm resolutions to carry on the scientific game according to certain rules or stipulations.” Nevertheless, Morris Cohen considers a priori principles as “methodologic or regulative principles which enable us to organize our factual knowledge” and as “expressive of the fundamental nature of things,” What light does this view throw upon the methodological discussions of Hutchison, Kaufmann, Mises, and Knight?
  2. “While the deductive method might be applicable to a simple and stationary condition of industry, it becomes valueless in face of the increasing complexity of the modern economic world.” What was John Neville Keynes’ reaction to this point of view?
  3. “Just as the same proposition may express both a universal and a historical, or both a verbal and a real judgment, so it may express both a positive and a normative judgment.” (Fraser, Economic Thought and Language). First explain each of the three sets of antonyms and then explain and illustrate the statement.

II.

  1. Contrast and compare the logical nature of introspectionism and sensationalism as expounded by Felix Kaufmann or Morris Cohen.
  2. Give a reasoned explanation of Kaufmann’s distinction between three meanings of probability, one “relating to empirical knowledge as such”, another relating “to synthetic propositions undecided in a given scientific situation,” and a third referring to “the relative frequency of an attribute” within a certain collective.
  3. Felix Kaufmann, having made a distinction between empirical laws and theoretical laws, states: “Whereas we have both types of laws in natural science, there are, as I see it, no empirical laws established in social science, and even the tendency to establish such laws is not very strong. But if we consider the significance of theoretical laws in natural science, we cannot regard this as constituting a fundamental difference between the methods of natural science and those of social science.” Explain and discuss this statement in a way intelligible to someone who has not read Kaufmann’s writings.

III.

  1. Explain what Ludwig von Mises means by ”methodological apriorism”, “methodological individualism”, and “methodological singularism”.
  2. “In the history of applied Economics, the work of a Jevons, a Menger, a Bowley, has much more claim on our attention than the work of, say, a Schmoller, a Veblen, or a Hamilton.” What is Robbins driving at with this [last word cut off, “statement?” matches the spacing of the tips to the letter “t” that are still visible]
  3. Hutchison implies that pure theory may help the analyst to formulate questions to be answered by empirical studies: “The constant object of the scientist…is to compel the facts of experience to answer his questions definitely ‘yes’ or ‘no’…” Robbins appears to reverse the relationship: “Realistic studies may suggest the problem to be solved….But it is theory and theory alone which is capable of supplying the solution.” Discuss the paradox from the point of view of any of the other writers on methodology.

IV.

  1. If the description or institutional part of economics is viewed by Professor Knight as lying in the domain of cultural anthropology rather than economics proper, does this mean that in institutional inquiries sense observation assumes greater emphasis than intercommunication and interpretation? If not, why does Knight distinguish institutional from theoretical economics?
  2. “There are no better terms available to describe the difference between the approach of the natural and the social sciences than to call the former objective and the latter subjective.” (Hayek, “Scientism and the Study of Society”.) Explain the meaning of the essential terms employed and the statement as a whole.
  3. Hayek said: “It is only in so far as some sort of order arises as a result of individual action but without being designed by any individual that a problem is raised which demands a theoretical explanation.” Explain.

V.

  1. “Economics is in fact the only science which enjoys the advantage of an automatic quantification of its subject matter.” (Parsons, “Sociological Elements in Economic Thought”). Explain and discuss.
  2. Parsons distinguishes the following ideal types of criticism of abstract economic theory: (1) supplementary positivistic empiricism: (2) radical positivistic empiricism; (3) romantic empiricism; (4) supplementary non-economic sociology. Characterize each in a brief statement illustrated by examples.
  3. Discuss Veblen’s principal categories of human action—especially the “pragmatic” versus the “workmanlike” type—and compare them with the general “rational” type and the narrower “economic” type used in the abstract theories of traditional economics.
  4. On what grounds do Professors Herskovits and Knight reject and defend, respectively, the concept of the “economic man” as a useful tool of economic analysis?

 

Source:  Johns Hopkins University. The Ferdinand Hamburger, Jr. Archives. Department of Political Economy, Series 6. Box 3/1. Folder: “Department of Political Economy, Graduate Exams, 1933-1965”.

Image Source:  Johns Hopkins University yearbook, Hullabaloo, 1957, p. 28.

Categories
Exam Questions M.I.T.

M.I.T. Midterm and Final exams. Income and Employment Theory. Domar, 1968-69

 

Previously posted are the outlines, readings, and exams for Domar’s national income and employment courses taught at the University of Chicago in 1948 and at M.I.T. in 1965. Also of interest here are the MIT student evaluations for this and other core theory courses in the late 1960s.

There is little doubt in my mind that MIT economics graduate students during the first term of the 1968-69 academic year responded to Evsey Domar’s attempts to get them interested in the details of national income and product accounting and productivity indexes with an enthusiasm to rival the high-school kids’ reaction to the Hawley-Smoot lesson attempted by a high-school teacher in the cult-film Ferris Bueller’s Day Off.

Fun-fact: the boring teacher in the movie was played by Ben Stein, son of economist, Herbert Stein. One more piece of fun: watch an older Ben Stein talk about getting that role.

In Domar’s defense, national accounting and index numbers have never been the stuff of a great TED talk. Anyone? … Anyone?

__________________

THE THEORY OF INCOME AND EMPLOYMENT
14.451
E. D. Domar

MIDTERM EXAMINATION
November 27, 1968

Seventy-five minutes

Please answer all questions. Note their weights. Use a separate book for each question.

  1. [30%] “One of the basic defects of the American economic system lies in the presence of a large number of persons who receive legal incomes and yet perform no useful services. The inclusion of their incomes in the national income total (according to accepted methods) undoubtedly exaggerates this total.”
    Discuss this statement carefully. Distinguish different kinds of income and different sources. In each case indicate how national income (or product) will be affected if these persons were employed productively. (What does “productively” mean in this context?)
  2. [30%] In national income comparisons between the Soviet Union and the U.S. (or other pairs of less developed and advanced countries) it is usually thought that existing methods of social accounting understate the ratio of Russian to American income, particularly if official rates of exchange are used for conversion.
    1. Evaluate this statement critically and indicate whether or not you agree with it and why.
    2. Suppose the comparison was made first in Russian prices (for both countries) and then in American prices (again for both countries). Which one should give the Russians a more favorable ratio and why? (Hint: consider comparisons over time in the same country.)Note: Disregard the complexities of the Russian price system: remember that the Russian ruble is not freely convertible into other currencies.
  3. [25%] a. What role or roles does the so-called “Money Illusion” play in the Classical and Keynesian systems?

b. If a country is suffering from inflation, will an increase in output brought about by the reduction in unemployment (assuming that it existed) intensify or reduce the inflation? Why? (Hint: this is not an easy question; consider carefully the nature of output to be produced.)

  1. [15%] If you wanted to measure labor or other factor productivity would you or would you not use the Federal Reserve Index of Industrial Production? Why or why not?

 

Source: Duke University. David M. Rubenstein Rare Book and Manuscript Library. Economists’ Papers Archives. Evsey D. Domar Papers. Box 17, Folder “Macroeconomics Examinations (1 of 3)”.

__________________

THE THEORY OF INCOME AND EMPLOYMENT
14.451
E. D. Domar

FINAL EXAMINATION
Jan. 28, 1969

Three hours

Please answer Question 1 and any FOUR out of the five remaining questions. Use a separate book for each question.

  1. [24%] An overheard argument among students in 14.451 about measures required to increase investment:

Student A: Increase the quantity of money in order to reduce the rate of interest.

Student B: No, an increase in the quantity of money will merely raise prices.

Student C: A fall in the rate of interest will not have any appreciable effect on investment in any case.

Student D: A fall in the rate of interest will reduce savings, and thus reduce rather than increase investment.

Student E: To increase investment we should increase savings by increasing income inequality.

Student F: On the contrary, greater income equality will increase savings.

Student G: You are both (That is, E and F) wrong: the fraction of income saved is independent of income distribution and of the size of a person’s income.

Student H: To increase investment we should increase demand by increasing consumption and thus reducing saving.

Student I: Nothing will help unless you reduce the prices of machinery and construction.

Student J: You (that is, Student I) are wrong: your suggestion will merely reduce the amount of investment.

Please set this poor, confused group straight (if you can). In so doing, explain clearly the assumptions and conditions implied in each statement and evaluate it critically. Try to identify the opera (and its author) from which each aria is taken. How would you go about increasing investment?

  1. [19%] “Thus the rate of interest is what it is because it is expected to become other than it is; if it is not expected to become other than it is, there is nothing left to tell us what it is…”
    1. Can you identify the author of this famous statement?
    2. Can you recognize whose interest theory he referred to?
    3. Explain and evaluate that theory critically.
    4. Present your own (original or otherwise) theory of interest.
  2. [19%] a. Assume that all expenditures on education and training, both private and public, are to be treated as investment. Explain the modifications that you would make in existing methods of national income, (and product) and wealth accounting and the reasons for these changes.

b. In computing national product, each commodity (or service) is multiplied by its price in order to compute the total. What is the rationale for this method? What assumptions is it based on? Are these assumptions realistic?

  1. [19%] Suppose Project I has a higher internal rate of return, while Project II has a larger discounted value. Assume that the projects are mutually exclusive, and that both are being considered by a private firm.
    1. Explain the rationale of each method and the assumptions it is based on.
    2. Which method (that is, the internal rate vs. present value) would you use under what conditions and why?
    3. How will your calculations be changed if the projects are undertaken by a government of some underdeveloped country?
  2. [19%] a. “A high ratio of depreciation to investment is a sign of old age.”

b. “If the measured distribution of income remained the same in the U.S. over the last fifty years, the distribution of permanent income has become less equal.”
Comment. Explain your conclusion thoroughly.
c. “If the Balanced-Budget Theorem is correct, is Say’s Law also correct?”
Comment. Explain what is meant by each part of this statement.

  1. [19%] a. “What is the proper definition of money required in the Price-Flexibility (Patinkin-like) problems? Why and how does it differ from the usual definition?
    Explain what elements of American money supply and of other relevant assets you would include or exclude in the proper (for this purpose) definition of money.

b. Explain how Patinkin’s conclusions regarding the effects of an increase (say, of doubling) in the quantity of money “by magic” on the price level and on the rate of interest are modified by the existence of the money illusion in the labor market.
c. Assume the absence of money illusion and explain why the effects of creating money by open market operations differ from those when money is created “by magic.”

 

Source:Duke University. David M. Rubenstein Rare Book and Manuscript Library. Economists’ Papers Archives. Evsey D. Domar Papers. Box 17, Folder “Macroeconomics Examinations (2 of 3)”.

Image Source: Evsey D. Domar at the MIT Museum.

Categories
Chicago Exam Questions

Chicago. Graduate Prelim Exam for International Trade, 1970

 

Determining authorship for a committee’s prelim exam is difficult. The fact that this copy of the exam was found in Lloyd Metzler’s papers is a sign that he likely had a hand in composing at least part of the exam. One can see an inconsistency in British/US spelling (labour vs. labor) that leads me to conclude that Harry Johnson was also likely a co-author.

___________________

INTERNATIONAL TRADE
Preliminary Examination for the Ph.D. and A.M. Degrees
Winter 1970

WRITE THE FOLLOWING INFORMATION ON YOUR EXAMINATION PAPER:

Your code number and NOT your name
Name of Examination
Date of Examination

(Write in Black Ink)

Results of the examination will be sent to you by letter.
Answer all questions. Time: 4 yours

  1. Answer question (a) or (b)
    1. Assume a Heckscher-Ohlin model economy in which one of the two products is the capital good, population is constant; and a certain fixed proportion of the existing capital stock wears out each year. The economy devotes a certain proportion of the value of its annual output to gross saving.
      1. Analyse the long-run equilibrium of the economy, in isolation from foreign trade.
      2. Analyse the effects of the opening of trade at fixed terms of trade on the economy’s long-run equilibrium.
      3. Comment on the implications of your analysis for the conflicting views that free trade is the best policy, and that tariffs promote economic development.
    2. Assume, in contrast to the Heckscher-Ohlin model, that while labour is mobile between the two industries capital is specific to its industry (and in the closed economy fixed in quantity in each industry).
      1. What can you conclude about the effects of the opening of free trade on factor prices, assuming factors immobile?
      2. How are these conclusions altered by the assumption that capital in one industry is internationally mobile but remains sector-specific (i.e. a certain stock of capital is confined to the automobile industry, but can locate in either “Canada” or “The United States”?
      3. What would be the effects of the imposition of a tariff on Canadian imports of automobiles, on the location of production and on factor prices?
  1. Answer question (a) or (b)
    1. Keynes argued that in a system of flexible exchange rates involving a forward market, the forward rate has a constitutional weakness of the demand side. Thus, he said that while there are many asset holders with foreign assets who would like to hedge by selling forward exchange, there are few holders of foreign liabilities who would like to hedge by purchasing forward exchange.
      1. Assuming that interest rates are the same at home as abroad, what does this imply with respect to the discount or premium of the forward rate, all rates being measured in terms of the domestic-currency price of the foreign currency?
      2. Discuss the validity of Keynes’ argument, first on the assumption that inter-market arbitrage exists, and second on the assumption that it does not.
    2. A given country produces two commodities, food and manufactures, with two factors, labour and land. Suppose that food is land-intensive in the sense that the optimal ratio between land and labour is higher than in manufactures for all factor price ratios. Suppose further, that the production functions for both commodities are homogeneous of the first degree so that increasing the inputs of labour and land by fifty per cent in any commodity, increases output, also by fifty per cent.
      1. Given fixed amounts of labour and land, prove that the product-substitution schedule has the characteristics of a diminishing returns schedule, despite the fact both food and manufactures are produced at constant cost.
      2. How do you account for this appearance of diminishing returns?
      3. Suppose that Country A has a larger land-labour ratio than Country B. Is it possible that A may nevertheless import food, the land-intensive commodity and export manufactures, the labor-intensive commodity? Indicate graphically how this may occur. Is this result inconsistent with the Heckscher-Ohlin theorem that international trade raises the prices of the low-cost factors and lowers the prices of the high-cost factors? Explain.
      4. Is it a possible explanation of the Leontief paradox, which shows that the United States exports labour-intensive commodities and imports capital-intensive commodities?
  1. Suppose the world is composed of two large blocs and a few other countries. Suppose that the two large blocs do not intervene in the exchange market. Analyze, in the context of the optimum currency area literature, the consideration which would persuade one of the outside countries to peg their currency to one rather than the other currency area.
  2. It has been shown by Mundell that if one factor is internationally mobile and a country imposes a tariff on imports, the result will be the termination of international trade. What happens if the country simultaneously imposes a tax on the earnings of the factor that moves?
  3. “The ‘Keynesian’ theory of devaluation developed by Joan Robinson, James Meade, A. C. Harberger, H. G. Johnson and others depends on the assumption of ‘money illusion’ on the part of the labour force. If that assumption is replaced by the assumption that wages in the long run are determined according to the theory of marginal productivity, a completely new theory of devaluation has to be developed.”
    Discuss this quotation, and if you agree with it sketch the nature of the new theory required.
  4. “The optimum tariff argument for protection is the only valid first-best economic argument for a tariff. All the other arguments are either second-best economic arguments, non-economic arguments, or non-arguments.”
    Discuss, giving examples. How would you describe the infant-industry argument?
  5. Discuss the main arguments for and against the following proposed solutions for the adjustment problem of the international monetary system:

(i) the “wider band”
(ii) the “crawling peg”
(iii) a rise in the price of gold

  1. Answer question (a) or (b)
    1. “The established version of the theory of effective protection is unsatisfactory because it attempts to combine a general equilibrium theory of demand with a partial-equilibrium theory of supply. If the usual Heckscher-Ohlin assumptions about production are made, the theory falls apart.”
      Discuss this quotation.
    2. or
      1. Discuss the controversy between Johnson and Metzler concerning the transfer problem under the conditions postulated by Keynes. (You need not indicate what you regard as the correct result but only what were the main points of the controversy.)
      2. What changes were made by Metzler in the orthodox or prevailing theory, generally but erroneously attributed to Ohlin? Show that these changes are in accord with Johnson’s “Suggestions for Simplifying Balance of Payments Theory.”

 

Source:Duke University. David M. Rubenstein Rare Book and Manuscript Library. Economists’ Papers Archive. Papers of Lloyd Appelton Metzler, Box 9, Folder “Exams 302”.

Image Source: Tariff reform–Cleveland and Thurman, ca. 1888  from Library of Congress Prints and Photographs Division Washington, D.C. 20540 USA

Categories
Exam Questions Harvard Suggested Reading Syllabus

Harvard. Economy of the U.S. Course outline, readings, exam. Leontief, 1945-46

 

 

Not much to say here about the material I have found for the first iteration of Wassily Leontief’s course on the economy of the United States other than I was surprised that his own book, Structure of the American Economy, 1919-1929, was not mentioned among the readings.

_______________

Course Announcement

Economics 12a. The Economy of the United States.

Half-course (fall term). Mon., Wed., and (at the pleasure of the instructor) Fri., at 3.  Associate Professor Leontief.

Source: Announcement of the Courses of Instruction offered by the Faculty of Arts and Sciences during 1945-46. Published in Official Register of Harvard University, Vol. 42, No. 8 (March 31, 1945), p. 36.

_______________

Course Enrollment

[Economics] 12a (fall term) Associate Professor Leontief.—The Economy of the United States.

Total 30: 2 Graduates, 8 Seniors, 5 Juniors 5 Sophomores, 10 Radcliffe.

Source:Harvard University. Report of the President of Harvard College, 1945-46, in Official Register of Harvard University, Vol. 45, No. 12 (May 20, 1948), p. 58.

_______________

ECONOMICS 12a
The Economy of the United States
Winter Semester, 1945-46

  1. General Interrelation of Industries and Households:
    1. Commodity flow and allocations of commodities and services.
    2. Cost structure of industries and Direct and Indirect demand.
    3. Capital stock and the Balance of Saving and Investment.
    4. Basic determinants of the Economic Structure of the United States: National Resources and Population, State of technical arts, consumers’ behavior, Public Policies
  2. Structural Characteristics of Selected Branches of the National Economy:
    1. Manufacturing
    2. Mining
    3. Agriculture
    4. Transportation
    5. Foreign Trade
    6. Domestic Trade
    7. Service Industries
  3. Structure of Consumers’ Demand:
    1. Sources of income
    2. Income distribution
    3. Spending and Saving pattern
  4. Price Structure:
    1. Price structure and the industrial structure
    2. Prices and incomes
  5. Economic Structure and Economic Policies

In this course, lectures are supplemented by simple research problems assigned as home work.

Readings:

J. R. Hicks and A. G. Hart, The Social Framework of the American Economy

and selected readings from publications of

National Resources Planning Board
National Bureau of Economic Research
Brookings Institution

[Handwritten additions:
Williamson, Growth of American Economy
Kuznets, Secular Movements of Production]

            Since Economics 12a is being given for the first time, the above outline probably will be modified in the course of instruction.

*  * *  *  *  *

Handwritten list following course outline

Econ 12A

Desk. Hicks JR & Hart—Social Framework

Desk. Leontief W—Economic statistics & postwar policies.
Reprint Harris Post-war Economic Problems.

Desk. National Resources Committee—Structure of the American Economy

Desk. National Resources Planning Board. Industrial Location & Nat. Resources

Desk. Kuznets, S.S.—Secular Movements in Production & Prices

Desk. Bell, S.—Productivity, wages and national income. Chs. 1, 3, 4, 5, 6, 7, 9. Appendix A: II & III

Desk. Glover, J.G.—Development of American Industries. Chs. 17, 31

Desk. Williamson, Growth of the American Economy. Chs 20, 21, 22.

Desk. U.S. Nat. Resources Comm., Consumer Spends his Income

Desk. Bd. Governors Federal Res. System [Postwar Economic Studies No. 1]. Jobs, production & living standards, 1945.
Goldenweiser)

 

Source:  Harvard University Archives. Syllabi, course outlines and reading lists in economics, 1895-2003 (HUC 8522.2.1), Box 4, Folder “Economics, 1945-1946 (1 of 2)”.

_______________

1945-46
HARVARD UNIVERSITY
ECONOMICS 12a

The Economy of the United States
Final. January. 1946.

Answer FOUR questions including question six. If you choose to answer question one, spend approximately one hour on it; in the final score it will be given double weight.

  1. Present a short discussion of your special research topic.
  2. How would you go about estimating the probable effect of a changed distribution of national income on the output and employment in the metal fabricating industry?
  3. Discuss the principal factors which have determined the changes in agricultural employment from the end of the last century up to the beginning of the second World War.
  4. Analyze the position of foreign trade in the structure of American economy.
  5. Describe the mutual dependence of wages, profits, and prices from the point of view of interindustrial relationships.
  6. Review critically of the papers included in the Postwar Economic Studies assigned for the Reading Period.

Source:  Harvard University Archives. Harvard University, Final examinations, 1853-2001. Box 11, “Harvard University, Faculty of Arts and Sciences. Papers Printed for Final Examinations. History, History of Religions, …, Economics, … , Military Science, Naval Science. January, 1946”.

Image Source:  Wassily Leontief in Harvard Class Album 1947.

Categories
Exam Questions Harvard Socialism Suggested Reading Syllabus

Harvard. Economics of socialism. Outline, readings, final exam. Schumpeter, 1943-44

 

Earlier Economics in the Rear-view Mirror posted the course outline and final examination for Joseph Schumpeter’s course on the economics of socialism that was given in the second semester of 1945-46. None of the final examination questions were shared between these two years, so together the exams provide a better idea of what was actually covered than either alone.

______________________

Course Announcement

Economics 11b. Economics of Socialism

Half-course (winter term). Mon., Wed., and (at the pleasure of the instructor) Fri., at  10. Professor Schumpeter.

 

Source: Announcement of the Courses of Instruction Offered by the Faculty of Arts and Sciences During 1943-44. Official Register of Harvard University, Vol. 40, No. 21 (September 29, 1943), p. 33.

______________________

Course Enrollment

[Economics] 11b (winter term) Professor Schumpeter. –Economics of Socialism.

Total 26:  3 Graduates, 5 Seniors, 2 Juniors, 1 Sophomore, 13 Navy.

 

Source: Harvard University. Report of the President of Harvard College and Reports of Departments for 1943-44, p. 56.

______________________

ECONOMICS 11b
1943-44
OUTLINE AND ASSIGNMENTS

  1. FIRST TWO WEEKS: The Socialist Issue.

Socialist ideas and socialist parties. Socialism and the labor movement. Laborite and intellectualist socialism. The Definition of Socialism.

H. W. Laidler*, History of Socialist Thought, 1927.
T.M. Sogge, “Industrial Classes in the U. S.  in 1930,” Journal of the American Statistical Association, vol. 28 (1933), pp. 199-203.
Encyclopaedia of the Social Sciences, article on Socialist and Labor Parties.

  1. THIRD TO FIFTH WEEK: The Theory of Centralist Socialism.

O. Lange and F. M. Taylor*, The Economic Theory of Socialism.
H. D. Dickinson, Economics of Socialism, 1939.

  1. SIXTH TO NINTH WEEK: The Economic Interpretation of History. The Class Struggle, and the Marxist Theory of Capitalism.

Karl Marx, Capital, Volume I, chs. I, IV, V, VI.
Marx and Engels, The Communist Manifesto
Paul M. Sweezy*, The Theory of Capitalist Development, 1942, chs. I-VI (pp. 1-108).

  1. TENTH TO TWELFTH WEEK: The Socialist Theory of the State and of the Proletarian Revolution, Imperialism, National Socialism.

V.I. Lenin, State and Revolution.
V. I. Lenin, Imperialism.
M. Dobb, Political Economy and Capitalism, ch. VII.
Paul M. Sweezy*, The Theory of Capitalist Development, Chs. XIII-XIX.

READING PERIOD ASSIGNMENT

Read E. Bernstein, Evolutionary Socialism, especially pp. 18-95, and survey again the items in the reading list marked *.

 

Source:Harvard University Archives. Syllabi, course outlines and reading lists in economics, 1895-2003 (HUC 8522.2.1). Box 3, Folder “Economics, 1943-1944 (2 of 2)”.

______________________

1943-44
HARVARD UNIVERSITY
ECONOMICS 11b
[Final. February, 1944]

One question may be omitted. Arrange your answers in the order of the questions.

  1. Describe briefly the emergence of either the English Independent Labour Party or the German Social Democratic Party.
  2. In the Second International, opinion was divided on the question whether socialists should or should not participate in bourgeois governments. What were the arguments that were adduced for and against? Which groups expressed the one and which the other view? Which view prevailed eventually within the Second International?
  3. What are the rules of rational allocation of productive resources in a socialist society, and how do they differ from the corresponding rules in a capitalist society (a) under conditions of perfect competition and (b) under conditions of monopolistic competition?
  4. State and criticize the Marxian proposition known as the Theory of Increasing Misery (“immiseration”).
  5. Most socialist writers recognize that the transition from the capitalist to the socialist form of life will raise a number of problems that are distinct from the problem of how to run a socialist society when established. What are those “transitional problems” and what methods have been suggested for dealing with them?
  6. What is meant by Reformism? By Revisionism? By Laborism?

 

Source:  Harvard University Archives. Harvard University. Final Examinations, 1853-2001. Box 8, Folder “Final examinations, Winter term, 1943-44”.

Image Source:  Harvard Class Album 1942.

Categories
Chicago Exam Questions Suggested Reading Syllabus

Chicago. Readings and Exam Questions for Graduate Money. Friedman, 1963

 

 

The reading list for Milton Friedman’s graduate money course, Economics 331, for the Winter Quarter of 1970 at the University of Chicago has been posted earlier. Here I have transcribed the (shorter) reading list from late 1963 along with the final exam questions and the take-home essay to be handed in on the day of the exam.

____________________

ECONOMICS 331—MONEY
Autumn Quarter, 1963

READING LIST

Milton Friedman

[NOTE: Readings marked with an asterisk (*) cover the essential substantive material.]

  1. Introductory Material

Milton Friedman*, The Quantity Theory, (forthcoming Encyclopaedia article)
D. H. Robertson, Money
David Hume, “Of Money,” “Of Interest,” in Essays and Treatises

  1. The Quantity Equation

Irving Fisher*, The Purchasing Power of Money (Macmillan, 1913), chaps. 1, 2, 3, 4, 8
A. C. Pigou*, “The Value of Money” in Readings in Monetary Theory [, Lutz, F. A., and Mints, L. W. (eds.)]
J. M. Keynes*, Tract on Monetary Reform (1924), chap. II; chap. III, Sec. I
Wesley C. Mitchell*, Business Cycles, The Problem and Its Setting (New York, 1927), pp. 128-39
Henry Thornton, An Enquiry into the Nature and Effect of the Paper Credit of Great Britain (1802), Library of Economics edition (Allen and Irwin, 1939), chaps. III and XI
Jacob Viner, Studies in the Theory of International Trade (Harpers, 1937), pp. 119-289
Alfred Marshall, Official Papers, “Evidence before the Indian Currency Committee (1889),” questions 11758-62 (pp. 267-69); “Evidence before the Gold and Silver Commission (1887-88).” questions 9629-86 (pp. 34-53); testimony to Royal Commission on The Depression of Trade and Industry (1886), answers to question 8(i), pp. 7-15

  1. The Demand for Money

Milton Friedman*, “The Quantity Theory of Money: A Restatement” in Studies in the Quantity Theory of Money, ed., M. Friedman
______________ “The Demand for Money: Some Theoretical and Empirical Results,” Journal of Political Economy (August, 1959), pp. 327-51
H. G. Johnson*, “Monetary Theory and Policy,” American Economic Review (June, 1962), Part II
W. J. Baumol, “The Transactions Demand for Cash: An Inventory Theoretic Approach,” Quarterly Journal of Economics (November, 1952)
James Tobin, “The Interest Elasticity of Transactions Demand for Cash,” Review of Economics and Statistics (August, 1956)
__________, “Liquidity Preference as Behavior Toward Risk,” Review of Economic Studies (August, 1956), pp. 241-47
J. M. Keynes*, The General Theory of Employment, Interest and Money, chaps. 13 and 15
J. R. Hicks*, “A Suggestion for Simplifying the Theory of Money,” Readings in Monetary Theory
Joan Robinson, “The Rate of Interest,” Econometrica, Vol. 19 (1951), reprinted as chap 1 of The Rate of Interest and Other Essays
Allan H. Meltzer, “The Demand for Money: The Evidence from the Time Series,” Journal of Political Economy (June, 1963)
[handwritten marginal note:
(Allan H. Meltzer, ) “The D. for M: A Cross Section Study of Bus Firms” Q.J.E., Aug. 1963]
Phillip Cagan*, “The Monetary Dynamics of Hyperinflation,” in Studies in the Quantity Theory of Money, esp. pp. 25-35 and 86-91
H. A. Latane, “Cash Balances and the Interest Rate—A Pragmatic Approach,” Review of Economics and Statistics (November, 1954) and (November, 1960)
James Tobin, “Liquidity Preference and Monetary Policy,” Review of Economics and Statistics, Vol. 19 (May, 1947), 130-31
Clark Warburton, “Monetary Velocity and Monetary Policy,” and Tobin’s rejoinder, Review of Economic Statistics, XXX (November, 1948), 310-17
John V. Deaver, “The Chilean Inflation and the Demand for Money,” unpublished Ph.D. dissertation (The University of Chicago, Department of Economics, Winter, 1961)
Edgar Feige, “The Demand for Liquid Assets: A Temporal Cross-Section Analysis,” unpublished Ph.D. dissertation (The University of Chicago, Department of Economics, Spring, 1963)
George R. Morrison, “Liquidity Preferences of Commercial Banks,” unpublished Ph.D. dissertation (The University of Chicago, Department of Economics, Winter, 1963)

  1. The Supply of Money

Milton Friedman and Anna J. Schwartz*, “Appendix B: Proximate Determinants of the Nominal Stock of Money,” from A Monetary History of the United States, 1867-1960 [copies on reserve]
H. G. Johnson*, “Monetary Theory and Policy,” Section III
Phillip Cagan, “The Demand for Currency Relative to the Total Money Supply,” Journal of Political Economy (August, 1958)
A. J. Meigs, Free Reserves and the Money Supply (University of Chicago Press, 1962)
William Dewald, “Free Reserves, Total Reserves, and Monetary Control,” Journal of Political Economy (April, 1963)
Lloyd W. Mints, A History of Banking Theory, pp. 9-12, 29-35, 217-22, 236-40, 247-57, 265-87
Milton Friedman, A Program for Monetary Stability, chapter 2
Knut Wicksell, “The Influence of the Rate of Interest on Prices,” Economic Journal, 171 (June, 1907), 213-20
Federal Reserve System: Purposes and Function
A. G. Hart, “The ‘Chicago’ Plan of Banking Reform,” Readings in Monetary Theory
George Tolley, “Providing for Growth of the Money Supply,” Journal of Political Economy (December, 1957), pp. 465-85

  1. Liquidity and Financial Intermediaries

Edward Simmons, “The Relative Liquidity of Money and Other Things,” Readings in Monetary Theory
Roland N. McKean*, “Liquidity and a National Balance Sheet,” Readings in Monetary Theory
Phillip Cagan*, “Why Do We Use Money in Open Market Operations,” Journal of Political Economy (February, 1958)
J. G. Gurley, “Liquidity and Financial Institutions in the Postwar Period,” Study Paper No. 14, Joint Economic Committee, January, 1960
H. Makower and J. Marschak, “Assets, Prices, and Monetary Theory,” Readings in Price Theory
J. G. Gurley and E. S. Shaw, Money in a Theory of Finance
Alvin Marty, “Gurley and Shaw on Money in a Theory of Finance,” Journal of Political Economy (February, 1961)

  1. The Monetary Standard and International Monetary Arrangements

Lloyd Mints*, Monetary Policy for a Competitive Society, chaps. 4 and 5
Milton Friedman*, “Commodity Reserve Currency” and “The Case for Flexible Exchange Rates,” Essays in Positive Economics
H. G. Johnson, International Trade and Economic Growth, chaps. 6 and 7
J. M. Keynes, Tract on Monetary Reform, chap. III, secs. 2, 3, 4; chaps. IV and V (*especially chap. III, sec. 2; chap. IV, sec. 2)
J. M. Keynes, “Economic Consequences of Mr. Churchill,” in Essays in Persuasion
Egon Sohmen, Flexible Exchange Rates (University of Chicago Press, 1961)
“Conditions of International Monetary Equilibrium.”* Session at 1962 meeting of American Economic Association, with papers by H. G. Johnson, Richard E. Caves, and Peter B. Kenen, and Discussion by J. Marcus Fleming, Harry C. Eastman, and J. Herbert Furth, American Economic Review (May, 1963), pp. 112-46

  1. The Process of Adjustment: Inflation, Business Cycles

Milton Friedman and Anna J. Schwartz*, “Money and Business Cycles,” Supplement to Review of Economics and Statistics (Feb., 1963), containing proceedings of Conference on Monetary Economics. Also, comments by H. Minsky, A. Okun, and C. Warburton
Clark Warburton, “The Misplaced Emphasis in Contemporary Business-Fluctuation Theory,” Readings in Monetary Theory
Friedman, “The Inflationary Gap,” in Essays in Positive Economics
Phillip Cagan, “The Monetary Dynamics of Hyperinflation,” Studies in the Quantity Theory of Money
Eugene M. Lerner, “Inflation in the Confederacy, 1861-65,” Studies in the Quantity Theory of Money
Arnold C. Harberger, “The Dynamics of Inflation in Chile,” in C. Christ, et al., Measurement in Economics (Stanford University Press, 1963)

[Handwritten note at the end of the section:
Reuben A. Kessel and Armen A. Alchian, “Effects of Inflation”, J.P.E., Dec. 1962]

____________________

ECONOMICS 331 – Autumn, 1963
M. Friedman

Problem for Reading Period

[Due on Final Exam 1:30 P.M., December 13, 1963.]]

In recent years, commercial banks have frequently complained about competition from other financial intermediaries, notably savings and loan associations, and have pressed the Reserve System to raise the maximum rate member banks are permitted to pay on time deposits.

At least one representative of savings and loan associations has argued that commercial banks are being extremely short-sighted, that they are not in any way harmed by the expansion of savings and loan associations, and are only hurting themselves by competing vigorously for time deposits and offering higher interest rates to get them. Indeed, the argument goes, the commercial banks would be wise to get out of the time deposit business altogether.

The fundamental constraint on the commercial banks, it is argued, is the total volume of reserves made available to them by the Federal Reserve System. True because of lower reserve requirements on time than on demand deposits, commercial banks can have larger total deposits if they expand the fraction which are in the form of time deposits. However, competition from financial intermediaries forces commercial banks to pay a rate of interest on time deposits that roughly matches earnings from them. And time deposits do absorb some reserves. Hence, with a given volume of reserves made available by the Fed, the expansion of time deposits reduces the aggregate amount of demand deposits, on which banks pay no interest and with respect to which they have no competitors. Each bank thinks it gets time deposits at the expense of financial intermediaries or other banks, but all banks together get them only at the expense of demand deposits.

Analyze this argument. Do so, first, on the assumption that the volume of reserves made available to commercial banks would be precisely the same whether the commercial banks did or did not offer time deposit facilities. Next, indicate whether this assumption is or is not plausible; if so, why; if not, why not and in what direction it is wrong.

____________________

ECONOMICS 331
Final Examination – Autumn 1963

M. Friedman
December 12, 1963

  1. Indicate briefly but specifically the key idea you got out of each of the following readings on the reading list.
    1. W. C. Mitchell, Business Cycles
    2. J. R. Hicks, “A Suggestion for Simplifying the Theory of Money”
    3. Phillip Cagan, “Why Do We Use Money in Open Market Operations”
    4. J. M. Keynes, Tract on Monetary Reform.
  2. “In the early history of our country there was a dearth of currency and specie. It was difficult to have cash on hand, especially when most of the specie was used to pay for imports.” (E. R. Taus, Central Banking Functions of the United States Treasury, 1789-1941, p. 22.)
    Discuss the economic meaning of these sentences. Do they make sense as they stand? If so explain. If not, can you suggest any interpretation of them that does make sense? In your answer, emphasize analysis, not economic history.
  3. In class, it was pointed out that (a) it is widely believed that “easy” money tends to make for low interest rates and “tight” money for high interest rates yet (b) in fact interest rates generally tend to be rising or high when the stock of money is expanding rapidly and to be falling or low when the stock of money is expanding slowly or declining, whether the comparison is made among countries at one point in time [e.g., currently, Brazil or Chile vs. the U.S.; Japan vs. Switzerland], or over time for one country [e.g., 20’s vs. 1929-33 in U.S.].
    1. Give the theoretical analysis underlying (a).
    2. Give a theoretical analysis to rationalize (b). Indicate whether this analysis is consistent with that given in (1.).
  4. Suppose U.S. Federal taxes are cut next year by an amount equal to $11 billion a year. Along strictly monetary theory lines, using as your framework the equation of exchange, analyze the effect on money income, prices, and interest rates under three alternative sets of circumstances:
    1. The cut in taxes is accompanied by an equal increase in the deficit, which is financed by increasing the stock of money at a rate of $11 billion a year more than it would otherwise have been increased.
    2. The cut in taxes is accompanied by an equal increase in the deficit, which is financed by borrowing from the general public with no effect on the stock of money.
    3. The cut in taxes is accompanied by an equal cut in expenditures, so there is no change in the deficit.

You are, of course, not expected to give quantitative answers but to indicate direction of effect and the economic parameters on which the magnitude of effect depends.

  1. Indicate the effect each of the following would have on the U.S. money supply under two alternative suppositions: A. The U.S. is on a gold standard; B. The U.S. is on a fiduciary standard with freely floating exchange rates.
    1. Increase in U.S. tariffs
    2. Increase in foreign tariffs
    3. Decline in legally required reserve ratio of commercial banks
    4. Rise in yield on productive investment and hence increase in demand for loanable funds.

 

Source:  Hoover Institution Archives. Papers of Milton Friedman. Box 77, Folder 8 “University of Chicago, Econ 331”.

Image Source:  Milton Friedman (undated) from University of Chicago Photographic Archive, apf1-06231, Special Collections Research Center, University of Chicago Library.